Difference between revisions of "1962 AHSME Problems/Problem 13"

m (Solution)
m (added answer)
Line 5: Line 5:
  
 
==Solution==
 
==Solution==
{{solution}}
+
<math> \boxed{B} </math>
 
 
 
==See Also==
 
==See Also==
 
{{AHSME 40p box|year=1962|before=Problem 12|num-a=14}}
 
{{AHSME 40p box|year=1962|before=Problem 12|num-a=14}}

Revision as of 22:09, 26 December 2015

Problem

$R$ varies directly as $S$ and inverse as $T$. When $R = \frac{4}{3}$ and $T = \frac {9}{14}$, $S = \frac37$. Find $S$ when $R = \sqrt {48}$ and $T = \sqrt {75}$.

$\textbf{(A)}\ 28\qquad\textbf{(B)}\ 30\qquad\textbf{(C)}\ 40\qquad\textbf{(D)}\ 42\qquad\textbf{(E)}\ 60$

Solution

$\boxed{B}$

See Also

1962 AHSC (ProblemsAnswer KeyResources)
Preceded by
Problem 12
Followed by
Problem 14
1 2 3 4 5 6 7 8 9 10 11 12 13 14 15 16 17 18 19 20 21 22 23 24 25 26 27 28 29 30 31 32 33 34 35 36 37 38 39 40
All AHSME Problems and Solutions

The problems on this page are copyrighted by the Mathematical Association of America's American Mathematics Competitions. AMC logo.png